Solving the hardest question of a British Mathematical Olympiad

Поділитися
Вставка
  • Опубліковано 28 вер 2024
  • Thanks to Nahian for the suggestion! This is a difficult factorial problem.
    British Mathematical Olympiad 2002-2003 problem 5
    bmos.ukmt.org....
    A Mathematical Olympiad Primer Paperback - 1 Aug. 2011
    www.amazon.co....
    Math Forum post
    mathforum.org/l...
    Math StackExchange post
    math.stackexch...
    Subscribe: www.youtube.co...
    Send me suggestions by email (address in video). I consider all ideas though can't always reply!
    Like many UA-camrs I use popular software to prepare my videos. You can search for animation software tutorials on UA-cam to learn how to make videos. Be prepared--animation is time consuming and software can be expensive!
    Why are there comments before the video is published? Get early access and support the channel on Patreon
    / mindyourdecisions
    If you buy from the links below I may receive a commission for sales. (As an Amazon Associate I earn from qualifying purchases.) This has no effect on the price for you.
    Show your support! Get a mug, a t-shirt, and more at Teespring, the official site for Mind Your Decisions merchandise:
    teespring.com/...
    My Books (US links)
    Mind Your Decisions: Five Book Compilation
    amzn.to/2pbJ4wR
    A collection of 5 books:
    "The Joy of Game Theory" rated 4.2/5 stars on 105 reviews
    amzn.to/1uQvA20
    "The Irrationality Illusion: How To Make Smart Decisions And Overcome Bias" rated 3.6/5 stars on 10 reviews
    amzn.to/1o3FaAg
    "40 Paradoxes in Logic, Probability, and Game Theory" rated 4.1/5 stars on 20 reviews
    amzn.to/1LOCI4U
    "The Best Mental Math Tricks" rated 4.4/5 stars on 25 reviews
    amzn.to/18maAdo
    "Multiply Numbers By Drawing Lines" rated 4.5/5 stars on 16 reviews
    amzn.to/XRm7M4
    Mind Your Puzzles: Collection Of Volumes 1 To 3
    amzn.to/2mMdrJr
    A collection of 3 books:
    "Math Puzzles Volume 1" rated 4.5/5 stars on 30 reviews
    amzn.to/1GhUUSH
    "Math Puzzles Volume 2" rated 4.5/5 stars on 10 reviews
    amzn.to/1NKbyCs
    "Math Puzzles Volume 3" rated 4.5/5 stars on 8 reviews
    amzn.to/1NKbGlp
    Connect with me
    My Blog: mindyourdecisi...
    Twitter: / preshtalwalkar
    Newsletter (sent only for big news, like a new book release): eepurl.com/KvS0r
    2017 Shorty Awards Nominee. Mind Your Decisions was nominated in the STEM category (Science, Technology, Engineering, and Math) along with eventual winner Bill Nye; finalists Adam Savage, Dr. Sandra Lee, Simone Giertz, Tim Peake, Unbox Therapy; and other nominees Elon Musk, Gizmoslip, Hope Jahren, Life Noggin, and Nerdwriter.

КОМЕНТАРІ • 1,1 тис.

  • @MindYourDecisions
    @MindYourDecisions  4 роки тому +804

    I am always excited and nervous when I post a video like this! One the one hand, I am thrilled to share a challenging mathematical proof. On the flip side, proofs require perfection and it is very challenging to make a video with no errors. You guys have great attention to detail, so if you see any mistakes, let me know! For major mistakes I will repost a corrected video; for minor mistakes/typos I will leave a note in a comment. Hope you enjoyed this problem!

    • @hardestmaths6975
      @hardestmaths6975 4 роки тому +8

      Hy sir i wana talk to you

    • @tentic
      @tentic 4 роки тому +6

      ily! and ur videos!!, they help me learn so much! (I have completed pre calc course and I'm in middle school lol)

    • @ilias-4252
      @ilias-4252 4 роки тому +28

      You wanted solutions on the positive integers so you didnt have to check a=0 at the start of the video. Very minor mistake.

    • @replicaacliper
      @replicaacliper 4 роки тому +2

      More challenging stuff like this please!

    • @jameswan2963
      @jameswan2963 4 роки тому +3

      13a=13+a please help me for the solution

  • @sounak5853
    @sounak5853 4 роки тому +2039

    "Pause the video if you would like to give this problem a try"
    Thanks for your concern, I'll just skip 10 seconds instead.

    • @Azimsajid1729
      @Azimsajid1729 4 роки тому +1

      ua-cam.com/video/l8S0p-91h7A/v-deo.html

    • @tfdtfdtfd
      @tfdtfdtfd 4 роки тому +46

      No need to pause.....the short interval provided was enough to put together the proof

    • @RosidinAli
      @RosidinAli 3 роки тому +4

      😅😅😅

    • @wushu1984
      @wushu1984 3 роки тому +33

      He should have said turn off your device for a week as you attempt to give this problem a try...

    • @mydkarthikmecharena9010
      @mydkarthikmecharena9010 5 місяців тому +1

      😂

  • @VinaySingh-jm8iw
    @VinaySingh-jm8iw 3 роки тому +29

    Great proof, but how would one know they needed to prove that b

    • @typingcat
      @typingcat 3 роки тому +13

      Probably with lots of previous experiences with similar "proof" questions that used this kind of boundary checking/contradiction tests.

    • @bigboboluager5003
      @bigboboluager5003 3 роки тому +9

      You don't know that. The way he presents this is not the way you start thinking about the problem. I started out, trying a few easy values like 1 and 2 for a, b and c. You immediatly get a contradiction so you can keep that in the back of your mind. Sometimes this will come in helpfull later on and sometimes it wont. If you have some practice you will learn more tricks, that you can just kind of throw at the problem and see what happens.

    • @thunderstorm8653
      @thunderstorm8653 3 роки тому +3

      Experience

  • @WOLLEY104
    @WOLLEY104 Рік тому +6

    I'm a GCSE student and these kinds of videos are always fun to watch. I love maths but never understood how people even know where to start with these questions. I lost track of what was going on like a minute into the video.

  • @AlbertEinstein49607
    @AlbertEinstein49607 2 місяці тому +1

    Hello, British Mathematical Olympiad here, thanks for reviewing this question. I did actually get this question right in the olympiad but there is a much more simple way!

  • @Macieks300
    @Macieks300 4 роки тому +6

    Nice puzzle. I solved it kind of similarly but when I got to a

    • @rorschachkovacs7867
      @rorschachkovacs7867 4 роки тому

      I did the same way that you do, and eventually you can get the conclusion that n=0 and m=1, just aplying that n! is even if n is equal or greater than 2

    • @latenter1428571
      @latenter1428571 Рік тому

      There is an easier way to show a contradiction. Once a < b < c, then b! divides a!*b!, b! divides b! and b! divides c!. But b! cannot divide a! Then c < b (that's not our case) or a = b.

  • @Timotheeee1
    @Timotheeee1 4 роки тому +40

    how does anyone even come up with this

    • @typingcat
      @typingcat 3 роки тому +6

      Drugs.
      Lots of drugs.

    • @nelser1160
      @nelser1160 3 роки тому +6

      For real, how would someone know what to prove at the start?

    • @user-qs3ih3ll5f
      @user-qs3ih3ll5f 3 роки тому

      miracle, I guess.

    • @rustemtehmezov9494
      @rustemtehmezov9494 3 роки тому +2

      @@nelser1160 So, when you solve so much of questions like these, it's easy to check these metods and get bounds.

    • @nelser1160
      @nelser1160 3 роки тому +1

      @@rustemtehmezov9494 I wonder how many I have to solve to get that lol

  • @benheideveld4617
    @benheideveld4617 4 роки тому +17

    And without using the Gougu theorem...

  • @helo3827
    @helo3827 4 роки тому +3

    You are my favorite youtuber,
    I never miss any of your videos

  • @JuanMataCFC
    @JuanMataCFC 4 роки тому +2

    actually amazing proof, i think u covered every possibility. i don't think i've ever seen contradiction used better than this before!

  • @meenashetty9782
    @meenashetty9782 3 роки тому +5

    Halfway through i stopped listening and started reading comments.

  • @pokmaster4475
    @pokmaster4475 4 роки тому +5

    This is one of the best math proofs, I have seen

  • @charlesaudet9035
    @charlesaudet9035 4 роки тому

    The part showing a=b may be simplified (Time 4.44 to 7.44). Instead of dividing by a!, divide by b!. The left-hand side will be an integer a! and the right-hand-side of the equality will be a fraction a!/b! + 1 + c!/b! (since a

  • @HollywoodF1
    @HollywoodF1 3 роки тому

    4:10 Proposing that we have not yet shown that b < c because we’ve yet to prove that a solution exists. I suppose, when including later steps that proceed similarly, the proof proceeds on the assumption that a solution exists.

  • @afbanales
    @afbanales 4 роки тому +6

    Your definition of wonderful is WAY different than my definition of wonderful!

    • @ABHISHEK-xm5mt
      @ABHISHEK-xm5mt 4 роки тому

      Power of belief, Dare to believe 👇
      ua-cam.com/video/ygc-B_OHz9w/v-deo.html

  • @abysslight2490
    @abysslight2490 4 роки тому +1

    You could’ve proven the fact that a=b much easier if you divided both sides of your equation by b! instead of a! at 4:53. This is because we already knew that a is less than or equal to b, so the case where they are not equal would mean aa (as previously shown), c!/a! is an integer, and 1 is obviously an integer, but since b>a, a!/b! is not an integer, thus a! isn’t an integer. And that’s a contradiction. It’s a lot simpler and quicker way to get to the same conclusion.

  • @isaacdeutsch2538
    @isaacdeutsch2538 Місяць тому

    We immediately notice that the equation is invariant under a swap of a and b, so we may assume, without loss of generality, that b is at least a. Now, we don't know that c is at least b. We know that a! divides c! (reducing everything modulo a!). We would like to know that b! divides c! as well, which is equivalent to c is at least b. So consider. Note that since a is at most b, a! is at most b!, so a! + b! is at most 2b!. Suppose c < b. Then c! < b!, so a! + b! + c! is at most 3b! < a!b! once a > 2, so the only solutions with c < b have a = 1, 2. Suppose a = 1. Then b! = 1 + b! + c!, so c! = -1, a contradiction. So a is not 1. Suppose a = 2. Then 2b! = 2 + b! + c!, so b! = 2 + c!. Since we're assuming c < b, reducing modulo c shows that c divides 2, so c = 1, 2. But 2 + 1! = 3 and 2 + 2! = 4, neither of which are factorials. So there are no solutions with c < b, thus c is at least b.
    Now at this point, we could do a more convoluted argument that I did first, but I realized a simplification to my argument as I type this up, so in the interest of my thumbs I will present this more streamlined argument*. Since c! is divisible by b!, reduce the original equation modulo b!. Then we get that a! = 0 (mod b!), so b! | a!. But then since a! | b!, a! and b! are associate, and since they're both positive integers, this means a! = b!. But then a = b. So we can reduce the original equation to a!² = 2a! + c!, and dividing everything by a!, we get a! = 2 + c!/a!.
    Suppose for the moment that a > 2. Then 3 | a!, so reducing modulo 3, we see that c!/a! - 1 is divisible by 3. Thus c!/a! is not divisible by 3, and thus c = a, c = a + 1, or c = a + 2. c = a means that a! = 3, impossible. c = a + 1 means that a! = a + 3, leave that for now. c = a + 2 means that a! = a² + 3a + 4. Reducing modulo a, we see that a | 4, so a = 1, 2, or 4. Since we are supposing that a > 2, a = 4, but that. polynomial at 4 yields the value 32, which is not a factorial. So the only option is c = a + 1, which entails solving a! = a + 3. Now, for x > 3, we have the following inequalities of functions: x! > 2x > x + 3, and thus since we've assumed a > 2, our equation can only hold if a = 3. Indeed, 3! = 6 = 3 + 3, so (3, 3, 4) is a possible solution--indeed it is, 3!3! = 3! + 3! + 4!. This is thus the only solution for a > 2.
    Finally, recall that if a = 1, we get a contradiction--we did not use the assumption c < b for that. So assume a = 2. Then we wish to solve b! = 2 + c! with no restrictions on b or c. Since we know there are no solutions when c < b, suppose not, and reduce modulo b. Then we see that b | 2, so b = 1, 2. If b = 1, we get c! = -1 again, impossible, and if b = 2, we get c! = 0, still impossible. Thus there are no solutions if a = 2 either, and the only solution in the positive integers is (3, 3, 4).
    *In my original argument, I reduced modulo 2 instead of modulo b!, which shows quickly that b = a or b = a + 1. I then treated these cases separately (obviously, from the argument I have presented, the case b = a + 1 ended in contradiction). Some interesting stuff came up, but it's not as elegant as reducing modulo b! and showing directly that a = b.

  • @justinmay2295
    @justinmay2295 4 роки тому +27

    I was so close to the answer

  • @CraigNull
    @CraigNull 3 роки тому

    Once you establish 3

  • @trnfncb11
    @trnfncb11 3 роки тому

    It is somewhat easier to prove that a=b, for if b>a, then a!=a!/b!+1+c!/b!. Here the 1st term on the rhs is 1/n < 1 and the last term is either integer (which wouldn't work) or 1/m. But then 1/n+1/m must be 1 so a=2 and 2b!=2+b!+c! or b!=c!+2 with no solution.

  • @hamster8706
    @hamster8706 4 роки тому +7

    Fun fact: A prime number in russian is "простое число", which can mean simple number too,therefore, a prime number is the opposite of a complex number

    • @yurenchu
      @yurenchu 4 роки тому

      _Prime number_ is the "opposite" of _composite number_ . A composite number is a (positive) integer that can be written as the product of two (positive) integers that are both not equal to 1 ; in other words, a composite number is _composed_ as the product of two (non-unit) integers.
      (For example, 35 is a composite number because 35 = 5 * 7 . Another example: 9 is a composite number because 9 = 3 * 3 .)
      The word _complex_ also means "composite"/"composed", but in a different way: a complex number is composed as the sum of a real number and an imaginary number. (Since the real number may be 0 and/or the imaginary number may be 0i, a complex number may also have just a real value, or just an imaginary value.)

  • @HagenvonEitzen
    @HagenvonEitzen 4 роки тому

    Wlog a>=b and right from the start use b! - 1 = b!/a! + c!/a!, note that on the right are only integers or numbers >0 and = b = a - or a!=2, b!=c!=1 (which doesn't work). So a!^2 = 2 a! + c!, or: a! - 2 = c!/a!. In particular, the right hand side is positive, so a > 2. As (2a)!/a! > a!, we see c

  • @mogol8359
    @mogol8359 3 роки тому +1

    it took him a little long to prove a=b, if you divide both sides to a, then rewrite the equation and divide both sides with b, the only solution would be a=b

  • @kritika1413
    @kritika1413 4 роки тому +21

    Thumbnail: solve
    Me : NO
    Because I can't 😂😂😂

    • @JohnDlugosz
      @JohnDlugosz 4 роки тому

      The numbers are small, so trial and error will take less than ten minutes.

    • @kaisarsihaloho975
      @kaisarsihaloho975 4 роки тому +2

      Thumbnail : solve
      Me : Yes
      Brain : Can't
      Me : No
      Presh : HAH! I KNEW IT!

  • @alekosbantias3962
    @alekosbantias3962 4 роки тому

    I didn't even try. I had to pause the video several times to digest the information. Wonderful indeed

  • @justinyoung6342
    @justinyoung6342 4 роки тому

    The next-to-last step where you showed ca, we know c=a+1. Further, we can see that c itself is congruent to 1 mod 3.

  • @michaelblankenau3129
    @michaelblankenau3129 Рік тому

    This problem is even tougher when you’re still grappling with why 2 is less then 3

  • @cowabunka
    @cowabunka 3 роки тому

    i find it amazing that people can derive this prove out of thin air
    I have trouble following it even as I am watching the solution
    and I wonder how can someone take the necessary steps and making the correct deduction without knowing what the solution / direction to the solution is
    well done for having this problem, made my day

  • @phoenixarian8513
    @phoenixarian8513 3 роки тому +1

    Let me think... (a!-1)(b!-1)=c!+1
    However the function of factorial is extremely exponential. Just try to find the correct c!+1 to equal to the multiply result of a!-1 and b!-1.
    function result of ! equals to 1,2,6,24,120,720,5040,40320,362880,3628800.
    I've found out 5*5=25. a=3, b=3, c=4. Don't know if there are other answers.

  • @facilvenir
    @facilvenir 4 роки тому +3

    It was hard enough to follow your explanation! Thank God I didn't even try to solve it myself!

  • @Temirman13
    @Temirman13 4 роки тому

    Not sure, but maybe there is a faster way. Assuming that m=b!/a! is more than 1 and c >b, we get that m divides b! and should thus divide 1+m+c!/a!. Considering the fact that C!/b! is divisible by m. Thus 1 should be divisible by m. Hence m=1

  • @Name-is2bp
    @Name-is2bp 4 роки тому +2

    Hello Presh, can you please add in the description of your videos the requirements of what math concepts we need to be familiar with to solve each problem??

  • @achaemenid
    @achaemenid 4 роки тому +2

    I luv ur videos!
    Keep up the great work!

  • @gavin.d.m
    @gavin.d.m 4 роки тому +1

    This channel reinvigorates my interest in maths. Generally I can follow the solution but this one lost me. But still, it's fascinating to watch. 👏😄

  • @jonathannagel7427
    @jonathannagel7427 3 роки тому

    Remind me why factorials are useful, besides math classes as an engineering student, I have never come across any situation where a “!” has been necessary. Maybe some CS fold can chime in with an application?

    • @carvoloco4229
      @carvoloco4229 3 роки тому

      For example, calculating permutations and combinations and, with them, probabilities

  • @rogersledz6793
    @rogersledz6793 3 роки тому

    Thank you so much for uploading this video. It is helping me to get through the pandemic!

  • @MASTER5537
    @MASTER5537 3 роки тому +1

    Maths exam: Find all solutions in positive integers... Show your steps.
    Me: Yes

  • @martinhuschenbett4190
    @martinhuschenbett4190 3 роки тому

    That's a very nice solution, in most parts a bit simpler than mine. Nevertheless, I think this one could be simplified even further. When trying to prove that a = b by deriving a contradiction from b! = 1! + b! / a! + c! / a!, we could argue as follows: b divides b! (obviously), b divides b! / a! (since a < b) and b divides c! / a! (since a < b < c). Thus b would need to divide 1 as well, which cannot be for b > 3.

  • @georgekh541
    @georgekh541 2 роки тому +1

    I actully solved this problem the same way you have but the part you proved a=b i did it in a way simpler way , i divided both sides by b!

  • @armacham
    @armacham 4 роки тому +3

    1:48 "suppose A is either zero or one" -- why would I suppose that A could be equal to zero, when the problem clearly states that the values of A, B, and C are limited to positive integers?

    • @theqwertyman9309
      @theqwertyman9309 4 роки тому +1

      For the sake of argument, I guess?

    • @yurenchu
      @yurenchu 4 роки тому

      In some languages/cultures/education systems, 0 is considered positive (as well as negative). Presh was just being rigourous.

  • @akiskarorimakis741
    @akiskarorimakis741 3 роки тому

    U dont need to check the case of a=0 since we're talking about positive integers, otherwise (2,2,0) would be also a solution

  • @jamesjames1549
    @jamesjames1549 3 роки тому

    Simetrical equations in most cases lead into result that those variables are actually equal, as in this case.

  • @vikaspandey3537
    @vikaspandey3537 4 роки тому +4

    Got it in first look

  • @satishpatidar9582
    @satishpatidar9582 4 роки тому

    Please make a video on epicycles and fourier transformation

  • @sougatanandy6404
    @sougatanandy6404 4 роки тому +1

    Finally a quality video from you. Was missing such videos ❤

    • @JSSTyger
      @JSSTyger 4 роки тому +1

      What a backhanded compliment...

  • @InfiniBandR
    @InfiniBandR 4 роки тому

    What a solution you are a Brilliant man, Maker of this problem must be a Genius.

  • @AlexLetizia
    @AlexLetizia 4 роки тому

    I would solve in a different way, considering the number of zeros at the end of a!b!. This goes as (a*b)/16 more or less, and the number of zeros at the end of a!+b!+c! is around min(a,b,c)/4, therefore if the minimum is c you get easily a contradiction, if the minimum is a (or b), you get that b (or a) has to be less than < 5 to avoid having LHS >> RHS. You can make easy considerations for the case b=3 and b=4 and find the solution.

    • @jonasdaverio9369
      @jonasdaverio9369 4 роки тому

      I don't see how considering c to be the minimum would give a contradiction. Why couldn't you have (a*b)/16 to be about the same as c/4 ?

  • @iamadooddood4331
    @iamadooddood4331 4 роки тому

    No need to test a = b = 4, c = 6. Since c!/a! is not divisible by 3, c cannot be a multiple of 3. Additionally c = a + 2 only works if a is a multiple of 3, otherwise c!/a! will be divisible by 3.

  • @SonnyBubba
    @SonnyBubba 2 роки тому +1

    We have a problem that is easy to check, but took several proofs-by-contradiction to solve.
    Does this have any bearing on the famous P=NP problem?

  • @vaiapatta8313
    @vaiapatta8313 2 роки тому

    Minor detail: You don't need to check the case a=0, since the description specifies positive integers.

  • @kirillsukhomlin3036
    @kirillsukhomlin3036 3 роки тому

    I find the step after noticing the symmetry completely unobvious and statements to prove-arbitrary. Starting with divisibility for me is much more obvious.

  • @lchen1144
    @lchen1144 3 роки тому

    At time 3:00, an alternative is to continue to let a=3, then c!=5b! - 6 = 4b! + (b! - 3!) > b! (Because b >=3), i.e., c > b.

  • @sohaibmalik8584
    @sohaibmalik8584 3 роки тому

    This problem is based only on assumptions. a,b, and c are just any random numbers.

  • @srinivasadireddi
    @srinivasadireddi 4 роки тому +3

    First

  • @joshuakahky6891
    @joshuakahky6891 3 роки тому +1

    *What stops a from being any multiple of 3 (other than 0)? I agree that the solution is unique, but how would you prove that?*

  • @kingofnumbers7660
    @kingofnumbers7660 2 місяці тому

    I just plugged things in and hoped I was lucky.

  • @TheNameOfJesus
    @TheNameOfJesus 3 роки тому

    At 1:37 I decided the hints would make it possible to solve by inspection. And ten seconds later, I did.

  • @raph2550
    @raph2550 4 роки тому +1

    I randomly stumbled upon the thumbnail of this video and tried to solve it.
    I did not find the solution yet, but I made some progress. I swear, I'll find it! And only then, I'll watch the video =D

  • @herogpi1
    @herogpi1 4 роки тому

    Another way is show that
    (a! - 1)*(b! - 1) = c! + 1
    Once we have that, we know that (c! + 1) doesnt have any factor below the c.
    So, we have that:
    c = 1 -> c! + 1 = 2 -> there is no "a" that (a! - 1) = 2
    c = 2 -> c! + 1 = 3 -> there is no "a" that (a! - 1) = 3
    c = 3 -> c! + 1 = 7 -> there is no "a" that (a! - 1) = 7
    c = 4 -> c! + 1 = 25 = 5*5 = (6-1)*(6-1) = (3!-1)*(3!-1) -> a = b = 3
    c = 5 -> c! + 1 = 121 = 11*11 -> there are no "a" and "b"
    c = 6 -> c! + 1 = 721 = 7*103 -> there are no "a" and "b"
    ...
    Now that we have one solution, we have to verify if there are more solutions for "c > 6". And that I don't know how to do!

  • @jamesjin1225
    @jamesjin1225 Рік тому

    I figured out 3, 3, 4 quite quickly but I can't prove whether it is the only solution or not.

  • @richardcuddy6166
    @richardcuddy6166 4 роки тому

    I found the unique solution at random assuming the case of a = b, then trying 0, 1, 2, 3, but couldn't prove it was the only case.

  • @georgechen8028
    @georgechen8028 4 роки тому +1

    The logic in solving this question is awesome.

  • @DavidAult
    @DavidAult 4 роки тому

    Very nicely done! Thank you for that!

  • @rontyson6118
    @rontyson6118 2 роки тому

    This is my second time watching this. Alas, I still couldn't prove 3,3,4 was only solution, lol. Great video btw.

  • @whyismyhandleshown
    @whyismyhandleshown 4 роки тому

    I think there is a logic error. Yes since 3

  • @chandrabhandari3348
    @chandrabhandari3348 4 роки тому

    I have a problem dude : Take a semi circle.then a rectangle whose length is less than diameter of circle and its based upon diameter of that semicircle at down and ( let's consider a side supppse right) at right its lower vertex and corner of semi circle make up and end ( end there touching each other) rectangles upper face intersects circle tangentially at its max point vertically and a line drawn from prev.corner to rectangles and circles intersection( not tangential) = 10 cm find the rectangles area.

    • @yurenchu
      @yurenchu 4 роки тому

      Answer is 50 cm² . The surprising thing is that the ratio (rectangle length)/(circle diameter) and/or the rectangle's aspect ratio (length:height) don't affect the answer.
      Let
      d = diameter of semi-circle
      L = length of rectangle (along the base)
      b = height of the (non-tangential) intersection between rectangle and semi-circle
      c = length of line from intersection to left bottom vertex of semi-circle
      α = angle between 10 cm line segment and base
      The area T of the triangle with sides d, c and 10 (which is a right-angled triangle, due to _Thales's theorem_ ) is given by
      T = b*d/2
      but also by
      T = 10*c/2
      ==> b*d = 10c [eq. 1]
      Furthermore, note that
      tan(α) = b/L = c/10
      ==> c = 10b/L [eq. 2]
      Plugging this result into eq. 1 gives
      b*d = 10*10b/L
      d = 100/L
      L*d = 100 [eq. 3]
      Required is area of rectangle, which is
      A = L * d/2 [eq. 4]
      Plugging the result of eq. 3 into this, gives
      A = 100/2 = 50 cm²

  • @theimmux3034
    @theimmux3034 4 роки тому +1

    This was really hard, I didn't even know how to start.

    • @ABHISHEK-xm5mt
      @ABHISHEK-xm5mt 4 роки тому

      Power of belief, Dare to believe 👇
      ua-cam.com/video/ygc-B_OHz9w/v-deo.html

  • @cookiesuprememperor7763
    @cookiesuprememperor7763 3 роки тому

    I understood all the steps and a very well done, but how am I supposed to do all of this in the first place

  • @mr.professor9066
    @mr.professor9066 4 роки тому

    My brain crazy to solve this problem

  • @ajvarninja415
    @ajvarninja415 3 роки тому +2

    This is like watching Sherlock Holmes solving a crime

  • @rb.arindam
    @rb.arindam 3 роки тому +1

    70% through the video.
    And I'm gonna go put my brain into a freezer then onto a vibrator before using it any further.

  • @vinitvsankhe
    @vinitvsankhe 2 роки тому

    The solution was NP complete... Easy to understand the verification steps but difficult to identify the verification steps.

  • @nigglefiggle
    @nigglefiggle 4 роки тому +40

    It’s not incredible.
    It’s *magnificent*

  • @xwtek3505
    @xwtek3505 Рік тому

    I have a different solution involving Wilson's theorem. Though it's easier to state the solution first before proving it.
    Define n = min(a,b,c)
    a, b >= 3 (look at the first part of the video)
    Thus a!b!, a! and b! = 0 (mod 3), thus c! = 0 (mod 3). So c>=3, and n>=3
    We know that there are only 2 values of a! mod (n+1)!, 0 and n!. And a!=0 mod (n+1)! iff a>=n+1. So
    a!b! =a!+b!+c! (mod (n+1)!)
    pn!n! = qn! (mod (n+1)!)
    pn! = q (mod (n+1))
    Where p = 1 if a=b=n, otherwise p=0
    And q is how many between a, b, and c equals n.
    According to Wilson's theorem, there are 3 situations to consider:
    a. n=3. This corresponds to
    2p=q (mod 4)
    Only p=1 and q=2 satisfies the equation, so a=b=3, corresponding to 3! 3! = 3! + 3! + 4!
    b. n+1 is composite >=6. This means q=n+1 (mod n+1). This means q>=n+1>=6, but there are only 3 variables
    c. n+1 is prime. This means p+q = 0 (mod n+1). The maximum value of p+q is 4. This shows that n=3, and n=3 results in n+1=4 being composite.

  • @babyli9205
    @babyli9205 4 роки тому +1

    Me before watching this video: You know, maybe if I work harder on Olympic maths I could actually qualify for my country’s Olympics!
    Me after watching this video: Ummm...or not...
    No but it actually was an incredible arithmetics problem!! Challenging to understand but SO FUN!!

  • @leonardotavares6607
    @leonardotavares6607 4 роки тому

    This problem was also on the 2019 Portuguese Mathematical Olympiads

  • @mysteryGuySaysHi
    @mysteryGuySaysHi 4 роки тому

    These mathematicians must be so cool

  • @ididntknowwhattowriteforth2240
    @ididntknowwhattowriteforth2240 3 роки тому

    Isn’t there like a region of math dedicated to these types of problems, its called number theory or something like that. Correct me if I’m wrong

  • @giabao576
    @giabao576 4 роки тому

    Great example of using contradiction!

  • @igoregalado5590
    @igoregalado5590 4 роки тому +1

    I died half way through the video. I lost too many brain cells. May I rest in peace.

  • @andreare7766
    @andreare7766 3 роки тому

    Has c!+1=0 got a solution in the complex plane? If not, we can define another type of numbers, something like complex factorial :D

  • @dankmeme5336
    @dankmeme5336 3 роки тому +2

    I tooke me two hours, but I solved it on my own

  • @alexandreman8601
    @alexandreman8601 4 роки тому +2

    1:19 Why would a be less than b, I don't get it.

    • @sutapadey5274
      @sutapadey5274 3 роки тому

      because there is no loss of generality

  • @miso-ge1gz
    @miso-ge1gz 4 роки тому

    I understand what he is doing but i would never in a milion years ever think about doing that

  • @rogueartist9419
    @rogueartist9419 4 роки тому

    make more number theory problems like this

  • @tarunmnair
    @tarunmnair 4 роки тому

    Amazing problem and amazing solution.

  • @trueriver1950
    @trueriver1950 3 роки тому

    4:00 yes you showed that b >= c leads to a contradiction.
    That does not prove b < c.
    It proves that iff a solution exists at all then b < c.
    In other words you ignore the possibility that there are no valid solutions. Later in the video you demonstrate that at least some solution exists; only then is it rigorous to assert b < c
    7:45 same criticism. We still don't know a solution exists at all

  • @aadsfawgwefaf7118
    @aadsfawgwefaf7118 3 роки тому

    What an incredible problem. I wonder how the Olympiad committee come up with these problems.

  • @gauravsankritya2331
    @gauravsankritya2331 4 роки тому

    Since some people are providing easy proof for a=b assuming c >= b, I am providing a proof behind that assumption. But it's worth mentioning that we have to prove c>=b. Here is the whole solution according to me to prove a=b.
    Let's denote original equation
    a!b! = a! + b! + c! -------- (1)
    a

  • @tilakshukla8962
    @tilakshukla8962 3 роки тому

    This can easily be solved by first dividing by a! then by b!. By doing this we get a=b . Then make a quadratic and solve.

    • @sadkritx6200
      @sadkritx6200 3 роки тому

      How bro? I don't quite understand. If u divide both sides by a! b! u get 1=1/a!+ 1/b!+ c !/a! b! . So even if u prove this video method halfway upto a=b and apply it the above equation u get 1=2/a!+ c! /a! ^2. How do u turn that into a quadratic?

    • @tilakshukla8962
      @tilakshukla8962 3 роки тому

      @@sadkritx6200 multiplying your equation by a!^2 then using shree dhara charya formula you get a!=1+root (1+c!)
      now keeping in mind c>a
      put some values of c.
      By putting 4 as the value of c we get a!=6, hence a=3.

    • @tilakshukla8962
      @tilakshukla8962 3 роки тому

      @@sadkritx6200 I did not say divide by a!b!. I said first divide by a! then separately divide by b! in another equation. From there we get that b!>=a! and a!>=b! . The inequalities can be true iff a=b.

  • @Maggy-5201
    @Maggy-5201 2 місяці тому

    I can understand every single steps in this proof, but how and who found it?
    Anyway, contradiction is powerful. I just recognized it in another minesweeper video.

  • @ArpanRayTV
    @ArpanRayTV 4 роки тому +2

    Power of programming 😂🎉:-
    import math
    for a in range(0,10):
    for b in range(0,10):
    for c in range(0,10):
    if math.factorial(a)*math.factorial(b)==math.factorial(a)+math.factorial(b)+math.factorial(c):
    print(a,",",b,",",c)
    Python programme for the question 😂😂

  • @ProfRonconi
    @ProfRonconi 3 роки тому

    What an amazing problem. I wonder who comes up with these mathematical jewels.

  • @maniacm1565
    @maniacm1565 3 роки тому

    "...But two does not divide fi-"

  • @AndroidFan238
    @AndroidFan238 Рік тому

    Amazing problem!

  • @firstnamelastname9806
    @firstnamelastname9806 3 роки тому

    In what this can be used in life, if you are not math professor?

  • @abgani4979
    @abgani4979 3 роки тому

    I wasn't born when this problem came in Olympiad

  • @andrewtakao897
    @andrewtakao897 3 роки тому

    At around 7:00... Why do all the terms need to be divisible by a+1 ?

  • @Redditard
    @Redditard 4 роки тому +1

    I understood this much 🤏

  • @nick012000
    @nick012000 4 роки тому

    Why does a>=3 and a|4 imply a=4? a could also be 8, 12, 16, 20, or any other number greater than 3 and divisible by 4.

    • @yurenchu
      @yurenchu 4 роки тому

      a|4 means "a is a divisor of 4" (or equivalently: "4 is divisible by a"). So a cannot be 8, 12, 16, 20, because 4 is not divisible by 8, 12, 16, 20 (the division would result in a fraction).